1

Dirichlet積分をしてみた

84
0
$$$$

今回の記事はめちゃくちゃ短いです.
タイトルにもある様にDirichlet積分をするだけです.

Dirichlet積分

次の事が成り立つ.
\begin{equation} \int_{t_1+t_2+\cdots+t_n=1\wedge t_1,t_2,...,t_n>0}t_1^{\lambda_1-1}t_2^{\lambda_2-1}\cdots t_n^{\lambda_n-1}(1-t_1-t_2-\cdots t_{n})^{\mu -1}dt_1dt_2\cdots dt_n=\frac{\Gamma(\lambda_1)\Gamma(\lambda_2)\cdots \Gamma(\lambda_n)\Gamma(\mu)}{\Gamma(\lambda_1+\lambda_2+\cdots\lambda_n+\mu)} \end{equation}

求める積分を$I_n(\lambda_1,\lambda_2,...,\lambda_n;\mu)$と置く.
$n=1$の場合はベータ関数となるので明らかに成り立つ.
\begin{eqnarray} I_1(\lambda_1;\mu)&=&\int_0^1t_1^{\lambda_1-1}(1-t_1)^{\mu-1}dt_1\\ &=&\frac{\Gamma(\lambda_1)\Gamma(\mu)}{\Gamma(\lambda_1+\mu)} \end{eqnarray}
そこで,一般の1,2,...,nまで成り立つと仮定する.そして,n+1の時を考える.
\begin{eqnarray} I_{n+1}(\lambda_1,\lambda_2,...,\lambda_n,\lambda_{n+1};\mu)&=& \int_0^1dt_1t_1^{\lambda_1-1}\int_{t_2+t_3+\cdots t_n+t_{n+1}=1-t_1 \wedge t_2,t_3,...,t_n,t_{n+1}>0}dt_2dt_3\cdots dt_ndt_{n+1}t_2^{\lambda_2-1}t_{3}^{\lambda_3-1}\cdots t_n^{\lambda_n-1}t_{n+1}^{\lambda_{n+1}-1}(1-t_1-t_2-\cdots t_n-t_{n+1})^{\mu-1}\\ &=& \int_0^1dt_1t_1^{\lambda_1-1}\int_0^{1-t_1}dt_2t_2^{\lambda_2-1}\int_{t_3+\cdots t_n+t_{n+1}=1-t_1-t_2 \wedge t_3,...,t_n,t_{n+1}>0}dt_3\cdots dt_ndt_{n+1}t_{3}^{\lambda_3-1}\cdots t_n^{\lambda_n-1}t_{n+1}^{\lambda_{n+1}-1}(1-t_1-t_2-\cdots t_n-t_{n+1})^{\mu-1}\\ &=&\cdots\\ &=& \int_0^1dt_1t_1^{\lambda_1-1}\int_0^{1-t_1}dt_2t_2^{\lambda_2-1}\cdots\int_0^{1-t_1-t_2-\cdots t_{n-1}}t_n^{\lambda_n - 1}\int_0^{1-t_1-t_2-\cdots -t_{n-1}-t_n}dt_{n+1}t_{n+1}^{\lambda_{n+1}-1}(1-t_1-t_2-\cdots -t_n-t_{n+1})^{\mu-1} \end{eqnarray}
より,$t_{n+1}=(1-t_1-t_2-\cdots -t_n)s$と置きなおすと,
\begin{eqnarray} \int_0^{1-t_1-t_2-\cdots-t_{n-1}-t_n}dt_{n+1}t_{n+1}^{\lambda_{n+1}-1}(1-t_1-t_2-\cdots -t_n-t_{n+1})^{\mu-1} &=& (1-t_1-t_2-\cdots-t_{n-1}-t_n)^{\lambda_{n+1}+\mu-1}\int_0^1s^{\lambda_{n+1}-1}(1-s)^{\mu-1}\\ &=& (1-t_1-t_2-\cdots-t_{n-1}-t_{n})^{\lambda_{n+1}+\mu-1}\frac{\Gamma(\lambda_{n+1})\Gamma(\mu)}{\Gamma(\lambda_{n+1}+\mu)} \end{eqnarray}
を得るので,これを先の式に代入して次式を得る.
\begin{eqnarray} I_{n+1}(\lambda_1,\lambda_2,...,\lambda_n,\lambda_{n+1};\mu)&=& \int_0^1dt_1t_1^{\lambda_1-1}\int_0^{1-t_1}dt_2t_2^{\lambda_2-1}\cdots \int_0^{1-t_1-t_2-\cdots -t_{n-1}}dt_nt_n^{\lambda_n-1}(1-t_1-t_2-\cdots -t_n)^{\lambda_{n+1}+\mu-1}\\ &=& I_{n}(\lambda_1,\lambda_2,...,\lambda_n;\lambda_{n+1}+\mu)\frac{\Gamma(\lambda_{n+1})\Gamma(\mu)}{\Gamma(\lambda_{n+1}\mu)}\\ &=& \frac{\Gamma(\lambda_1)\Gamma(\lambda_2)\cdots \Gamma(\lambda_n)\Gamma(\lambda_{n+1}+\mu)}{\Gamma(\lambda_1+\lambda_2+\cdots \lambda_n+\lambda_{n+1}+\mu)}\frac{\Gamma(\lambda_{n+1})\Gamma(\mu)}{\Gamma(\lambda_{n+1}+\mu)}\\ &=& \frac{\Gamma(\lambda_1)\Gamma(\lambda_2)\cdots \Gamma(\lambda_n)\Gamma(\lambda_{n+1})\Gamma(\mu)}{\Gamma(\lambda_1+\lambda_2+\cdots \lambda_n+\lambda_{n+1}+\mu)} \end{eqnarray}
よって,$n+1$の場合も成り立つことが示せたので,本定理は示された.

投稿日:20231124
更新日:20231124

この記事を高評価した人

高評価したユーザはいません

この記事に送られたバッジ

バッジはありません。

投稿者

ただ趣味で数学をやっている普通の人です。 特殊な知識もなくただ数学を楽しみたいenjoy勢です。正直間違った事も平気で書くかもしれません。 僕の書いている記事で間違いを発見した時は遠慮なくご指摘してくださると助かります。

コメント

他の人のコメント

コメントはありません。
読み込み中...
読み込み中